LSAT and Law School Admissions Forum

Get expert LSAT preparation and law school admissions advice from PowerScore Test Preparation.

User avatar
 Dave Killoran
PowerScore Staff
  • PowerScore Staff
  • Posts: 5852
  • Joined: Mar 25, 2011
|
#49541
Complete Question Explanation
(The complete setup for this game can be found here: lsat/viewtopic.php?t=18147)

The correct answer choice is (C)

Answer choice (A): From the last rule T and V are not consecutive, so this answer choice is wrong.

Answer choice (B): From the third rule S must arrive at some time before Y, and so this answer choice is incorrect.

Answer choice (C): This is the correct answer choice.

Answer choice (D): The second rule stipulates that W or Y is fourth, and so this answer choice is incorrect.

Answer choice (E): The fourth rule indicates that T arrives at some time after R, and so this answer choice is incorrect.

Get the most out of your LSAT Prep Plus subscription.

Analyze and track your performance with our Testing and Analytics Package.